Diễn Đàn MathScopeDiễn Đàn MathScope
  Diễn Đàn MathScope
Ghi Danh Hỏi/Ðáp Thành Viên Social Groups Lịch Ðánh Dấu Ðã Ðọc

Go Back   Diễn Đàn MathScope > Sơ Cấp > Việt Nam và IMO > 2014

News & Announcements

Ngoài một số quy định đã được nêu trong phần Quy định của Ghi Danh , mọi người tranh thủ bỏ ra 5 phút để đọc thêm một số Quy định sau để khỏi bị treo nick ở MathScope nhé !

* Nội quy MathScope.Org

* Một số quy định chung !

* Quy định về việc viết bài trong diễn đàn MathScope

* Nếu bạn muốn gia nhập đội ngũ BQT thì vui lòng tham gia tại đây

* Những câu hỏi thường gặp

* Về việc viết bài trong Box Đại học và Sau đại học


Trả lời Gởi Ðề Tài Mới
 
Ðiều Chỉnh Xếp Bài
Old 28-02-2014, 08:46 PM   #136
namdung
Administrator

 
Tham gia ngày: Feb 2009
Đến từ: Tp Hồ Chí Minh
Bài gởi: 1,343
Thanks: 209
Thanked 4,066 Times in 778 Posts
Gửi tin nhắn qua Yahoo chát tới namdung
Trích:
Nguyên văn bởi hothevinh View Post
Thay nam dung cho em hỏi 45 bạn thi TST 2013 co bạn nào giải được bài 6 không ạ và thầy có nhận xét gì về bài 6 năm đó ạ
Năm đó không ai đạt điểm tối đa bài này.Tôi đánh giá bài 6 là một bài hay.
[RIGHT][I][B]Nguồn: MathScope.ORG[/B][/I][/RIGHT]
 
namdung is offline   Trả Lời Với Trích Dẫn
Old 01-03-2014, 11:53 PM   #137
trungno
+Thành Viên+
 
Tham gia ngày: Nov 2012
Bài gởi: 32
Thanks: 79
Thanked 1 Time in 1 Post
Mình xin góp 1 bài:
2 đống sỏi:đống 1 có a viên;đống 2 có b viên.2 người chơi trò chơi như sau:mỗi người đến lượt của mình lấy 1 số sỏi tùy ý từ 1 đống hoặc lấy từ cả 2 đống với số sỏi bằng nhau.người bốc viên sỏi cuối cùng là người chiến thắng.hãy tìm tất cả các giá trị (a;b) để người đi sau có chiến thuật để thắng.
[RIGHT][I][B]Nguồn: MathScope.ORG[/B][/I][/RIGHT]
 
__________________
trungno is offline   Trả Lời Với Trích Dẫn
Old 02-03-2014, 01:08 AM   #138
huynhcongbang
Administrator

 
huynhcongbang's Avatar
 
Tham gia ngày: Feb 2009
Đến từ: Ho Chi Minh City
Bài gởi: 2,413
Thanks: 2,165
Thanked 4,188 Times in 1,381 Posts
Gửi tin nhắn qua Yahoo chát tới huynhcongbang
Trích:
Nguyên văn bởi trungno View Post
Mình xin góp 1 bài:
2 đống sỏi:đống 1 có a viên;đống 2 có b viên.2 người chơi trò chơi như sau:mỗi người đến lượt của mình lấy 1 số sỏi tùy ý từ 1 đống hoặc lấy từ cả 2 đống với số sỏi bằng nhau.người bốc viên sỏi cuối cùng là người chiến thắng.hãy tìm tất cả các giá trị (a;b) để người đi sau có chiến thuật để thắng.
Bài này là 1 dạng Nim, cụ thể là trò chơi Wythoff. Mình có post tại đây 1 lần và đáp số có nêu trong đó.

http://forum.mathscope.org/showthread.php?t=41750
------------------------------
Trích:
Nguyên văn bởi hothevinh View Post
Anh huynhcongbang có dap an de thi tst trung quoc năm 2013 với năm 2011 không nếu có anh post lên nhe
Anh không có em à.
Đề China TST thì em tham khảo nội dung trên mathlink ấy. Trên đó người ta cũng có giải khá đầy đủ, có bài nào thắc mắc thì em cứ trao đổi trực tiếp trên đó hoặc post lên MS để mọi người cùng thảo luận thôi.


[RIGHT][I][B]Nguồn: MathScope.ORG[/B][/I][/RIGHT]
 
__________________
Sự im lặng của bầy mèo

thay đổi nội dung bởi: huynhcongbang, 02-03-2014 lúc 01:13 AM Lý do: Tự động gộp bài
huynhcongbang is offline   Trả Lời Với Trích Dẫn
The Following User Says Thank You to huynhcongbang For This Useful Post:
trungno (02-03-2014)
Old 03-03-2014, 12:35 AM   #139
huynhcongbang
Administrator

 
huynhcongbang's Avatar
 
Tham gia ngày: Feb 2009
Đến từ: Ho Chi Minh City
Bài gởi: 2,413
Thanks: 2,165
Thanked 4,188 Times in 1,381 Posts
Gửi tin nhắn qua Yahoo chát tới huynhcongbang
Trích:
Nguyên văn bởi hothevinh View Post
Anh huynhcongbang anh có bài kiểm tra và bài giảng của trường xuân toán học hà nội năm 2014,2013,2012 không nếu có anh post lên cho em nhé
Các tài liệu đó thì chỉ có các bạn trực tiếp tham gia mới có được thôi em à. Em có quen ai thì hỏi xin thử chứ anh thì không có rồi.

Dưới đây là đề thi và đáp án đề trường Xuân của TPHCM 2014. Mọi người tham khảo thử nhé!


[RIGHT][I][B]Nguồn: MathScope.ORG[/B][/I][/RIGHT]
 
Hình Kèm Theo
Kiểu File : png Capture.PNG (25.9 KB, 200 lần tải)
File Kèm Theo
Kiểu File : pdf De Kiem Tra Truong Xuan 2014.pdf (213.8 KB, 307 lần tải)
Kiểu File : pdf Dap An De Kiem Tra Truong Xuan 2014.pdf (522.8 KB, 508 lần tải)
Kiểu File : pdf De Bai Warm Up.pdf (226.5 KB, 343 lần tải)
Kiểu File : pdf Dap An De Warm Up.pdf (297.5 KB, 327 lần tải)
__________________
Sự im lặng của bầy mèo
huynhcongbang is offline   Trả Lời Với Trích Dẫn
The Following 12 Users Say Thank You to huynhcongbang For This Useful Post:
buivantuanpro1 (06-07-2016), chuyentoanltt (04-03-2014), dangvip123tb (04-03-2014), dung_toan78 (03-03-2014), giabao185 (03-03-2014), greg_51 (04-07-2014), hoangqnvip (03-03-2014), keodua123 (14-03-2014), quocbaoct10 (04-03-2014), sang_zz (03-03-2014), thiendieu96 (03-03-2014), trungno (08-03-2014)
Old 13-03-2014, 12:57 AM   #140
kien10a1
+Thành Viên+
 
kien10a1's Avatar
 
Tham gia ngày: Feb 2011
Đến từ: Vĩnh Yên- Vĩnh Phúc
Bài gởi: 371
Thanks: 43
Thanked 263 Times in 153 Posts
Gửi tin nhắn qua Yahoo chát tới kien10a1
Tồn tại hay không một dãy số $a_n $ nguyên dương, tăng ngặt mà ước nguyên tố lớn nhất của ${a_n}^2+1 $ là một số nguyên tố p cố định.
[RIGHT][I][B]Nguồn: MathScope.ORG[/B][/I][/RIGHT]
 
__________________
Quay về với nơi bắt đầu
kien10a1 is offline   Trả Lời Với Trích Dẫn
The Following User Says Thank You to kien10a1 For This Useful Post:
hoangqnvip (13-03-2014)
Old 13-03-2014, 10:25 PM   #141
huynhcongbang
Administrator

 
huynhcongbang's Avatar
 
Tham gia ngày: Feb 2009
Đến từ: Ho Chi Minh City
Bài gởi: 2,413
Thanks: 2,165
Thanked 4,188 Times in 1,381 Posts
Gửi tin nhắn qua Yahoo chát tới huynhcongbang
Trích:
Nguyên văn bởi kien10a1 View Post
Tồn tại hay không một dãy số $a_n $ nguyên dương, tăng ngặt mà ước nguyên tố lớn nhất của ${a_n}^2+1 $ là một số nguyên tố p cố định.
Nếu hỏi thế này thì có thể phân tích 1 tí như bên dưới:

Xét thử $p=2$. Nếu có một dãy thỏa mãn đề bài thì phương trình $x^2+1=2^y$ có vô số nghiệm.
Tuy nhiên, điều này là không thể do $x$ lẻ và $(x-1)(x+1)=2(2^{y-1}-1)$, buộc $x=1,y=1$.

Do đó, có thể dự đoán câu trả lời là phủ định.


[RIGHT][I][B]Nguồn: MathScope.ORG[/B][/I][/RIGHT]
 
__________________
Sự im lặng của bầy mèo
huynhcongbang is offline   Trả Lời Với Trích Dẫn
Old 14-03-2014, 01:23 AM   #142
kien10a1
+Thành Viên+
 
kien10a1's Avatar
 
Tham gia ngày: Feb 2011
Đến từ: Vĩnh Yên- Vĩnh Phúc
Bài gởi: 371
Thanks: 43
Thanked 263 Times in 153 Posts
Gửi tin nhắn qua Yahoo chát tới kien10a1
Trích:
Nguyên văn bởi huynhcongbang View Post
Nếu hỏi thế này thì có thể phân tích 1 tí như bên dưới:

Xét thử $p=2$. Nếu có một dãy thỏa mãn đề bài thì phương trình $x^2+1=2^y$ có vô số nghiệm.
Tuy nhiên, điều này là không thể do $x$ lẻ và $(x-1)(x+1)=2(2^{y-1}-1)$, buộc $x=1,y=1$.

Do đó, có thể dự đoán câu trả lời là phủ định.

Nếu mà đưa p lên lớn hơn thì nó có thể có nhiều ước nguyên tố mà anh, lúc đó thì cũng hơi không chắc...
[RIGHT][I][B]Nguồn: MathScope.ORG[/B][/I][/RIGHT]
 
__________________
Quay về với nơi bắt đầu
kien10a1 is offline   Trả Lời Với Trích Dẫn
Old 14-03-2014, 02:07 PM   #143
huynhcongbang
Administrator

 
huynhcongbang's Avatar
 
Tham gia ngày: Feb 2009
Đến từ: Ho Chi Minh City
Bài gởi: 2,413
Thanks: 2,165
Thanked 4,188 Times in 1,381 Posts
Gửi tin nhắn qua Yahoo chát tới huynhcongbang
Trích:
Nguyên văn bởi kien10a1 View Post
Nếu mà đưa p lên lớn hơn thì nó có thể có nhiều ước nguyên tố mà anh, lúc đó thì cũng hơi không chắc...
À, thực ra nội dung anh post ở trên chỉ là dự đoán thôi.
Đúng là câu trả lời là phủ định, có thể tham khảo nội dung của 2 bài báo này:

http://www.emis.de/journals/AMI/2004/acta2004-luca.pdf

Trong đó có phát biểu 1 câu:
Nếu $P(x^2+1) < K$ thì $x < \exp ( \exp (O(k \log_2 k / \log k)))$, trong đó kí hiệu $P(n)$ là ước nguyên tố lớn nhất của $n$.

Như thế ứng với mỗi số nguyên tố $p$ cho trước thì số lượng $x$ sao cho $P(x^2+1)=p$ là hữu hạn, tức là không tồn tại dãy tăng ngặt vô hạn như đề bài yêu cầu.

Một số bài báo khác về vấn đề gpf (greatest prime factor) này:

Định nghĩa và tính chất:
http://mathworld.wolfram.com/GreatestPrimeFactor.html

Về số có dạng $(ab+1)(bc+1)(ca+1)$:
http://matwbn.icm.edu.pl/ksiazki/aa/aa79/aa79110.pdf

Về số nguyên tố Mersen hay rộng hơn là số có dạng $2^n-1$:
https://www.math.dartmouth.edu/~carlp/PDF/murata4.pdf
http://www.renyi.hu/conferences/erdo...es/stewart.pdf

Về số có dạng $n^2-1$:
http://www.math.leidenuniv.nl/~fnajman/FLFNMC.pdf

Về số có dạng $ab+1$:
http://arxiv.org/pdf/1311.1161v2.pdf

Về số có dạng $n!+1$:
https://www.renyi.hu/~p_erdos/1976-27.pdf


[RIGHT][I][B]Nguồn: MathScope.ORG[/B][/I][/RIGHT]
 
__________________
Sự im lặng của bầy mèo

thay đổi nội dung bởi: huynhcongbang, 14-03-2014 lúc 02:10 PM
huynhcongbang is offline   Trả Lời Với Trích Dẫn
Old 14-03-2014, 03:56 PM   #144
quocbaoct10
+Thành Viên Danh Dự+
 
quocbaoct10's Avatar
 
Tham gia ngày: Oct 2012
Đến từ: THPT chuyên Lê Quý Đôn-Nha Trang-Khánh Hòa
Bài gởi: 539
Thanks: 292
Thanked 365 Times in 217 Posts
bài 47: Cho $n$ là số nguyên dương tập hợp các bộ số sau:
$S_n=\{(a_1,a_2,...,a_{2^n})|a_i= 0; 1, \forall i=\overline{1,n} \}$.
Với 2 phần tử $a=(a_1,a_2,...,a_{2^n})$ và $b=(b_1,b_2,...,b_{2^n})$, đặt $d(a,b)=\sum\limits_{i=1}^{2^n}|a_i-b_i|$.
Tập con $A$ của $S$ là tập tốt nếu $d(a,b) \ge 2^{n-1}$ với mọi cặp $a,b$ phần tử phân biệt của $A$. Hỏi một tập con tốt của $S_n$ có thể có nhiều nhất bao nhiêu phần tử.
[RIGHT][I][B]Nguồn: MathScope.ORG[/B][/I][/RIGHT]
 
__________________
i'll try my best.
quocbaoct10 is offline   Trả Lời Với Trích Dẫn
Old 15-03-2014, 11:03 AM   #145
huynhcongbang
Administrator

 
huynhcongbang's Avatar
 
Tham gia ngày: Feb 2009
Đến từ: Ho Chi Minh City
Bài gởi: 2,413
Thanks: 2,165
Thanked 4,188 Times in 1,381 Posts
Gửi tin nhắn qua Yahoo chát tới huynhcongbang
Trích:
Nguyên văn bởi quocbaoct10 View Post
bài 47: Cho $n$ là số nguyên dương tập hợp các bộ số sau:
$S_n=\{(a_1,a_2,...,a_{2^n})|a_i= 0; 1, \forall i=\overline{1,n} \}$.
Với 2 phần tử $a=(a_1,a_2,...,a_{2^n})$ và $b=(b_1,b_2,...,b_{2^n})$, đặt $d(a,b)=\sum\limits_{i=1}^{2^n}|a_i-b_i|$.
Tập con $A$ của $S$ là tập tốt nếu $d(a,b) \ge 2^{n-1}$ với mọi cặp $a,b$ phần tử phân biệt của $A$. Hỏi một tập con tốt của $S_n$ có thể có nhiều nhất bao nhiêu phần tử.
Có một bài thế này:

Cho dãy hữu hạn có độ dài $n$ và các phần tử là 0 hoặc 1. Khoảng cách giữa hai dãy chính là số vị trí mà phần tử tương ứng ở hai dãy khác nhau. Xét $m$ dãy tùy ý và khoảng cách giữa hai dãy bất kỳ đều không vượt quá $d$. Chứng minh rằng $m\le \frac{2d}{2d-n}$.

Bài này có thể giải bằng cách dùng double counting. Tuy nhiên, ở đây thú vị là $2d=n$ nên đánh giá ở trên trở thành vô ích vì $m \le + \infty$. Mời các bạn góp ý thêm.
[RIGHT][I][B]Nguồn: MathScope.ORG[/B][/I][/RIGHT]
 
__________________
Sự im lặng của bầy mèo
huynhcongbang is offline   Trả Lời Với Trích Dẫn
Old 15-03-2014, 01:02 PM   #146
quocbaoct10
+Thành Viên Danh Dự+
 
quocbaoct10's Avatar
 
Tham gia ngày: Oct 2012
Đến từ: THPT chuyên Lê Quý Đôn-Nha Trang-Khánh Hòa
Bài gởi: 539
Thanks: 292
Thanked 365 Times in 217 Posts
Trích:
Nguyên văn bởi huynhcongbang View Post
Có một bài thế này:

Cho dãy hữu hạn có độ dài $n$ và các phần tử là 0 hoặc 1. Khoảng cách giữa hai dãy chính là số vị trí mà phần tử tương ứng ở hai dãy khác nhau. Xét $m$ dãy tùy ý và khoảng cách giữa hai dãy bất kỳ đều không vượt quá $d$. Chứng minh rằng $m\le \frac{2d}{2d-n}$.

Bài này có thể giải bằng cách dùng double counting. Tuy nhiên, ở đây thú vị là $2d=n$ nên đánh giá ở trên trở thành vô ích vì $m \le + \infty$. Mời các bạn góp ý thêm.
Em thấy bài 47 nó khá gần với bài toán này: "Chứng minh rằng không thể có nhiều hơn 4096 cây nhị phân độ dài 24 sao cho 2 cây bất kì trong chúng có ít nhất 8 vị trí khác nhau". Mò mẫm một hồi thì ra đáp án là $2^{n+1}$ nhưng vẫn chưa biết giải thích sao.
[RIGHT][I][B]Nguồn: MathScope.ORG[/B][/I][/RIGHT]
 
__________________
i'll try my best.
quocbaoct10 is offline   Trả Lời Với Trích Dẫn
Old 15-03-2014, 02:07 PM   #147
mathandyou
Moderator
 
Tham gia ngày: Dec 2012
Đến từ: HCMUS
Bài gởi: 557
Thanks: 259
Thanked 402 Times in 216 Posts
Mình xin đóng góp một bài sau:
Bài 48:Cho $\triangle ABC$ nội tiếp $(O)$.$D$ là điểm chính giữa cung $BC$ không chứa $A$.Kẻ đường kính $DE$.$BE$ cắt đường trung trực của $AD$ tại $K$.Qua $E$ kẻ đường thẳng vuông góc với $AC$ cắt $KO$ tại $P$.$AP$ cắt $KD$ tại $N$.Chứng minh:$K,B,P,N$ đồng viên và $N$ thuộc $(O)$.
[RIGHT][I][B]Nguồn: MathScope.ORG[/B][/I][/RIGHT]
 
__________________
Xét cho cùng, phần thưởng cao quý nhất mà công việc mang lại không phải là thứ bạn nhận được, mà nó vẽ nên chân dung con người bạn ra sao.

[Only registered and activated users can see links. ]

thay đổi nội dung bởi: mathandyou, 15-03-2014 lúc 04:03 PM
mathandyou is offline   Trả Lời Với Trích Dẫn
Old 15-03-2014, 06:17 PM   #148
Manhnguyen
+Thành Viên+
 
Tham gia ngày: Feb 2014
Đến từ: TPHCM
Bài gởi: 92
Thanks: 26
Thanked 29 Times in 28 Posts
Trích:
Nguyên văn bởi mathandyou View Post
Mình xin đóng góp một bài sau:
Bài 48:Cho $\triangle ABC$ nội tiếp $(O)$.$D$ là điểm chính giữa cung $BC$ không chứa $A$.Kẻ đường kính $DE$.$BE$ cắt đường trung trực của $AD$ tại $K$.Qua $E$ kẻ đường thẳng vuông góc với $AC$ cắt $KO$ tại $P$.$AP$ cắt $KD$ tại $N$.Chứng minh:$K,B,P,N$ đồng viên và $N$ thuộc $(O)$.
Theo cách làm của em thì mấu chốt nằm ở Cm $AB\perp BP$
Để Cm điều đó ta làm theo các bước:
-Kẻ đường cao $EL$ xuống $AC$, $OQ$ xuống $AD$
-Cm $\triangle OQD$ đồng dạng $\triangle ELC$
-Cm $\triangle BQD$ đồng dạng $\triangle BLC$
-Cm $\triangle BQL$ đồng dạng $\triangle BDC$
-Cm $ALQB$ nội tiếp=>$AB\perp BP$
$KD$ cắt $(O)$ tại $N'$
Ta Cm:
-Cm $\triangle PBK$ đồng dạng $\triangle ABD$
-Cm $\triangle PAB$ đồng dạng $\triangle KDB$
-$A,N',P$ thẳng hàng=>$N$ trùng $N'$=>đpcm
Từ đó dễ dàng Cm được $PBKN$ nội tiếp
[RIGHT][I][B]Nguồn: MathScope.ORG[/B][/I][/RIGHT]
 
__________________
Cần phải học, học nữa, học mãi
Suy nghĩ, chăm chỉ dẫu đúng sai
Tôi tư duy tức tôi tồn tại
Quyết tâm, cố gắng nên thiên tài.
Manhnguyen is offline   Trả Lời Với Trích Dẫn
Old 15-03-2014, 06:54 PM   #149
mathandyou
Moderator
 
Tham gia ngày: Dec 2012
Đến từ: HCMUS
Bài gởi: 557
Thanks: 259
Thanked 402 Times in 216 Posts
Thật ra anh chế bài này là từ bài:[Only registered and activated users can see links. ]
Và cái hình của nó khi anh chế đây:
Tuy nhiên cách giải của chú giải chắc cũng klq lắm.

[RIGHT][I][B]Nguồn: MathScope.ORG[/B][/I][/RIGHT]
 
Hình Kèm Theo
Kiểu File : jpg TST.jpg (44.1 KB, 296 lần tải)
__________________
Xét cho cùng, phần thưởng cao quý nhất mà công việc mang lại không phải là thứ bạn nhận được, mà nó vẽ nên chân dung con người bạn ra sao.

[Only registered and activated users can see links. ]
mathandyou is offline   Trả Lời Với Trích Dẫn
The Following User Says Thank You to mathandyou For This Useful Post:
thiendieu96 (16-03-2014)
Old 17-03-2014, 06:05 PM   #150
Traum
Moderator
 
Traum's Avatar
 
Tham gia ngày: Nov 2007
Đến từ: cyber world
Bài gởi: 413
Thanks: 14
Thanked 466 Times in 171 Posts
Trích:
Nguyên văn bởi quocbaoct10 View Post
bài 47: Cho $n$ là số nguyên dương tập hợp các bộ số sau:
$S_n=\{(a_1,a_2,...,a_{2^n})|a_i= 0; 1, \forall i=\overline{1,n} \}$.
Với 2 phần tử $a=(a_1,a_2,...,a_{2^n})$ và $b=(b_1,b_2,...,b_{2^n})$, đặt $d(a,b)=\sum\limits_{i=1}^{2^n}|a_i-b_i|$.
Tập con $A$ của $S$ là tập tốt nếu $d(a,b) \ge 2^{n-1}$ với mọi cặp $a,b$ phần tử phân biệt của $A$. Hỏi một tập con tốt của $S_n$ có thể có nhiều nhất bao nhiêu phần tử.
Google với từ khóa plotkin bound sẽ có nhiều thứ hay ho về vấn đwè trên đó.
[RIGHT][I][B]Nguồn: MathScope.ORG[/B][/I][/RIGHT]
 
__________________
Traum is giấc mơ.
Traum is offline   Trả Lời Với Trích Dẫn
The Following User Says Thank You to Traum For This Useful Post:
quocbaoct10 (17-03-2014)
Trả lời Gởi Ðề Tài Mới

Bookmarks

Ðiều Chỉnh
Xếp Bài

Quuyền Hạn Của Bạn
You may not post new threads
You may not post replies
You may not post attachments
You may not edit your posts

BB code is Mở
Smilies đang Mở
[IMG] đang Mở
HTML đang Tắt

Chuyển đến


Múi giờ GMT. Hiện tại là 01:25 AM.


Powered by: vBulletin Copyright ©2000-2024, Jelsoft Enterprises Ltd.
Inactive Reminders By mathscope.org
[page compression: 118.12 k/134.90 k (12.44%)]